24, 43, 89, 15 Which two numbers are prime numbers?

Answers

Answer 1

Answer: 43 and 49

Step-by-step explanation:

Answer 2

Answer:

43, 89

Step-by-step explanation:

A prime number is any number above 1 that can evenly divide out.

Hope This Helps :)


Related Questions

QUESTION 11
(01.02)
Given the function f(x) = 3(x+2) - 4, solve for the inverse function when x = 2. (1 point)
O -4
Oo
O 4
08

Answers

Answer:

8

Step-by-step explanation:

3( 2 + 2 ) - 4

3( 4 ) - 4

12-4

8

write an expression that represents the total amount of fencing needed to enclose the two fields. -how would you do this?-

Answers

Answer:

8x + 60

Step-by-step explanation:

4(x + 5) + 2(x - 10) + 2(x + 30)

= 4x + 20 + 2x - 20 + 2x + 60 (combine like terms)

= 8x + 60

The dimensions of a rectangular prism are length is 2 1/4 feet width is 1 foot and height is 1 1/4 feet and the length of the sides of a small cube are 1/4 foot each. How many small cubs can be packed in the rectangular prism.

Answers

Answer:

  180

Step-by-step explanation:

2 1/4 ft = 9/4 ft, so is a length equivalent to 9 cubes.

1 ft = 4/4 ft, so is a length equivalent to 4 cubes.

1 1/4 ft = 5/4 ft, so is a length equivalent to 5 cubes.

Then the volume of the prism is ...

  V = LWH

  V = (9)(4)(5) = 180 . . . cubes

What does x equal?
2(3x - 2) = 20

Answers

Answer:

4

Step-by-step explanation:

2(3x-2)=20

3x-2=10

3x=12

x=4

Answer:

x=4

Step-by-step explanation:

scarlett bought 10 chicken wings for $24.00. how much would it cost for 17 wings

Answers

First, divide.

24 / 10 = $2.40 per chicken wing

Second, multiply.

17 * 2.4 = $40.80

Best of Luck!

Is 22 + 32 = 42 a true statement?

Answers

False because 22 + 32 =54

Answer:

No. 22+32=54

Step-by-step explanation:

Which angle refers to the same angle as ∠ D E F

Answers

Answer:

It is angle FED which is equal to angle DEF

one-half the cube of m subtracted by 4 *

Answers

Answer:

[tex]\frac{m^3}{2} - 4[/tex]

Step-by-step explanation:

Required

Determine an expression for the given statement

The expression can be rewritten as:

one-half    *     the cube of m       subtracted by      4

Convert each statement to mathematical expressions:

One half = [tex]\frac{1}{2}[/tex]

The cube of m = [tex]m^3[/tex]

one-half    *     the cube of m = [tex]\frac{1}{2} * m^3[/tex]

At this stage, we have:

[tex]\frac{1}{2} * m^3[/tex] subtracted by 4

This gives:

[tex]\frac{1}{2} * m^3 - 4[/tex]

[tex]\frac{m^3}{2} - 4[/tex]

Hence, The corresponding expression is [tex]\frac{m^3}{2} - 4[/tex]

One-half the cube of m subtracted by 4 is [tex]0.5\sqrt[3]{m} -4[/tex]

Based on the information given, one-half the cube of m subtracted by 4 is expressed as:

[tex]0.5\sqrt[3]{m} -4[/tex]

Note that the cube of m is expressed as [tex]\sqrt[3]{m}[/tex]The half of the cube of m will be given as [tex]0.5\sqrt[3]{m}[/tex]

Subtracting 4 from the result will give the final expression as [tex]0.5\sqrt[3]{m} -4[/tex]

Learn more on expressions here: https://brainly.com/question/25968875

Put these into least to greatest

Answers

Answer:

Square root 8 over three, (square root of 4)/2, and pi/3.

Step-by-step explanation:

First of all, square the radicals to make solving this easier. Squarer root 8 over three will become 8/9, and square root 4 over 2 will now become 4/4, which is 1. Pi/3 is greater than 1, so we can now order this from least to greatest. 8/9 is the least, 1 is the second greatest value, and pi/3 is the greatest value.

Answer: 8squared/3 4squared/2 Pi/3

Step-by-step explanation:

8 squared = 2.8... = 2.8/3 (least)

4 squared = 2 = 2/2 = 1 (second least)

Pi over 3 = 3.14159.../3 = 1 .14159/3 (greatest)

Given h(x)= 4x^2, find h(3)

Answers

Answer: 36

Explanation:

h(x) = 4x^2

Find h(3):

Simply plug x = 3 in the function:
h(3) = 4(3)^2
h(3) = 4(9)
h(3) = 36
You can make x 3 in this to help get the answer. h(3)= 4(3)^2. Using the order of operations, we do 3^2 first. The answer for that is 9. Then we do 9 times 4, which is 36. 36 is our final answer. Hope this helps!

What is 90% of 81?
X

Answers

Answer:72.9

Step-by-step explanation:

Answer:

72.9

Step-by-step explanation:

Fractions method:

[tex]81*\frac{90}{100}[/tex] because 90/100 is the same thing as 90%

That gives us 7290/100, from 81*90/100

Simplify: =729/10 = 72.9

Decimals method:

81*9 = 729, add in decimal and that's 72.9

Cross-multiplication:

90    100

?         81

This is read as 90 to 100, (as a ratio) so how many to 81? To solve, cross-multiply - 81*90=7290, then divide by 100 to get 72.9.

This is my first answer , hope it helped!

PLEASE HELP ASAP Write the following equation in general form.
-x - 5y = -19

–x - 5y + 19 = 0
x + 5y - 19 = 0
x + 5y + 19 = 0

Answers

Answer:  -x - 5y + 19 = 0, choice A

All we're doing is adding 19 to both sides to get everything on the same side.

General form is Ax+By+C = 0. Some books use Ax+By = C which is just a variation of the same idea.

Point E is on line segment DF. Given DE = 2x, EF = 2x – 6, and
DF = 3x + 5, determine the numerical length of EF.

Answer: EF =

Answers

Answer:

EF is 16.  

Step-by-step explanation:

So we know that Point E is somewhere on Line DF. In other words, DE plus EF must equal DF. In an equation, this is:

[tex]DF=DE+EF[/tex]

We are given that DE is 2x, EF is 2x-6, and DF is 3x+5. So, substitute:

[tex](3x+5)=(2x)+(2x-6)[/tex]

Now, solve for x.

On the right, combine like terms:

[tex]3x+5=4x-6[/tex]

Subtract 4x from both sides:

[tex]-x+5=-6[/tex]

Subtract 5 from both sides:

[tex]-x=-11[/tex]

Divide both sides by -1:

[tex]x=11[/tex]

Now that we know x is 11, substitute this back into the equation for EF to find EF.

[tex]EF=2x-6[/tex]

Substitute 11 for x:

[tex]EF=2(11)-6[/tex]

Multiply:

[tex]EF=22-6[/tex]

Subtract:

[tex]EF=16[/tex]

And we're done!

Please help me! I am so bad at math. 7(6+5x)=3x-22

Answers

42+35x=3x–22
42+35x–3x=–22
42+32x=–22
32x=–22–42
32x=–64
x=–64/32
The answer is x= –2

Answer:

-2

Step-by-step explanation:

42+35x=3x-22

35x-3x=-22-42

32x=-64

therefore, x= -64/32

and through further simplification you will get

-2

hope this helped you ( ꈍᴗꈍ)

Solve the inequation (2x+3x)4 using distributive method.

Answers

Answer:

First, distribute the 4.

2x times 4  = 8x

3x times 4 = 12x

then add/ combine like terms.

8x + 12x = 20x

Step-by-step explanation:

Answer:

20x

Step-by-step explanation:

distribute the 4 to the 2x and 3x: 4 * 2x= 8x and 4 * 3x= 12x

then add the x's together: 8x+12x=20x

so the answer is 20x

What is the ratio for 2 red paper clips to 6 blue paper clips? Write it as a fraction.

Answers

Answer:

2 over 6 as a fraction simplifies as 1 over three.

Step-by-step explanation:

If you are writing as a fraction, you put 2 over 6 and simplify. Your answer is 1 over 3

Answer:

2/6  2/6  2/6  2/6  2/6  2/6  2/6  2/6  2/6  2/6  2/6  2/6  2/6  2/6  2/6  2/6  2/6  2/6  2/6  2/6  2/6  2/6  2/6  2/6  2/6  2/6  2/6  2/6  2/6  2/6  2/6  2/6  2/6  2/6  2/6  2/6  2/6  2/6  2/6  2/6  2/6  2/6  2/6  2/6  2/6  2/6  2/6  2/6  2/6  2/6  2/6  2/6  2/6  2/6  2/6  2/6  2/6  2/6  2/6  2/6  2/6  2/6  2/6  2/6  2/6  2/6  2/6  2/6  2/6  2/6  2/6  2/6  2/6  2/6  2/6  2/6  2/6  2/6  2/6  2/6  2/6  2/6  2/6  2/6  2/6  2/6  2/6  2/6  2/6  2/6  2/6  2/6  2/6  2/6  2/6  2/6  2/6  2/6  2/6  2/6  2/6  2/6  

Step-by-step explanation:

its just 2/6

why and how do you do this I am having a hard time

Answers

Answer:

Peter walks at a rate of 13/4 miles per hour.

Step-by-step explanation:

Peter walks at a rate of 13/4 miles per hour because every hour, represented by x, he walks 3.25 hours or 13/4. y is the total miles he walked so if he walks two hours the y would equal 6.5 or 26/4 and the x would equal 2.

Convert the given amount to the given unit.
27 yd; ft
27 yd = ft

Answers

Answer:

81 ft

Step-by-step explanation:

1 yard=3ft

27 x 3=81

So the answer is 81 ft

That would be 81 feet

In a stationary store,pencils have one price and pens have another price.Two pencils and three pens cost 78 cents.However, 3 pencils and 2 pens cost 72 cents . How much does one pencil cost?

Answers

Answer:

12¢

Step-by-step explanation:

Let the unit price of a pencil be x and the unit price of a pen be y.

2x+3y=78¢

3x+2y=72¢

(2x)+3y=78¢

(3x)+2y=72¢

=2x=78¢-3y

=2x/2=78¢/2+3y/2

=x=36¢-1.5y

You then use the equation 3x+2y=72¢.

3(36¢-1.5y)+2y=72¢

=117¢-4.5y+2y=72¢

=117¢-72¢=4.5y-2y

=45¢=2.5y

=45¢/2.5=y

=18¢=y

So a pen costs 18¢.

You then use x=36¢-1.5y

x=36¢-1.5(18¢)

=x=36¢-27¢

=x=12¢

Therefore, a pencil costs 12¢.

PLEASE HELP, ITS TIMED SO HURRY PLEASE>>>>>>>

Answers

Answer:

The answer is noncollinear

Step-by-step explanation:

Answer:

Non-collinear because the points dont line up to be colinnear. learned this today dead a** its not hard. pay attention

Step-by-step explanation:

Which statement is true? 2.289 < 2.297 2.297 < 2.289 3.01 < 2.297 2.289 > 3.01​

Answers

Answer:

The first one, 2.2289<2.297

Step-by-step explanation:

Answer:

The answer is 2.2289<2.297 because if you look in the hundredths place there is a 9 in 2.297 but in 2.2289 there is an 2 and 9>2 so that is your answer.

solve for x: 4/x=5/10

Answers

simplify 5/10 to 1/2
1. 4/x = 1/2

multiply both sides by x
2. 4= 1/2 x

simply 1/2x to x/2
3. 4= x/2

multiply both sides by 2
4. 4x2= x


simplify 4 x 2 to 8
5. 8= x

switch sides
x= 8

ANSWER; x= 8

Answer:

x = 8

Step-by-step explanation:

4/x = 5/10

=> 4/x = 1/2

=> 4 x 2 = x

=> 8 = x

=> x = 8

which of the following sets represents a function​

Answers

Answer:

the top one

Step-by-step explanation:

one input cannot have more than one output

Frank makes x dollars per week. His dad makes 4 times as much money as him each week. Each week their income adds up to 1.000 dollars. How much does frank earn each week? How much does franks dad earn each week?

Answers

Answer:

Frank earns $200.

His dad earns $800.

Step-by-step explanation:

Giving the following information:

His dad makes 4 times as much money as him each week. Each week their income adds up to 1.000 dollars.

We know that:

x= Franks income

4x= Dads income

y= total income

y= x + 4x

1,000= 5x

1,000/5=x

200=x

Frank earns $200.

His dad earns $800.

10
Write the numbers in order from least to greatest (**HINT: You need to
make them all decimals or all fractions***): 5/1000, 0.003, 9/1000

Answers

Answer:

0.003, 5/1000, 9/1000

Step-by-step explanation:

0.003 is thousandths.

5/1000 is thousandths.

9/1000 is thousandths.

I converted them to fractions.

5/1000 is already a fraction

Same for 9/1000

0.003=3/1000

So 3/1000, 5/1000, 9/1000

What is the greatest four-digit integer that meets the following three restrictions? 1- All of the digits are different. 2- The greatest digit is the sum of the other three digits. 3- The product of the four digits is divisible by 10 and not equal to zero.

Answers

I think that the greatest number that is all of these things is:9432

AG bisects ∢RAZ , and ∢RAG =. 8x+5 , and ∢ZAG = 4 x + 29 find the x .

Answers

Answer:

x = 6

Step-by-step explanation:

Given AG bisects ∠ RAZ , then

∠ RAG = ∠ ZAG , that is

8x + 5 = 4x + 29 ( subtract 4x from both sides )

4x + 5 = 29 ( subtract 5 from both sides )

4x = 24 ( divide both sides by 4 )

x = 6

1.5y-9.82=-9.37-10.1y-14.37 solve for y

Answers

Step-by-step explanation:

1.5y - 9.82 = -9.37 - 10.1y - 14.37

1.5y + 10.1y = 9.82 -9.37 - 14.37

11.6y = −13.92

y = (-13.92)/11.6

y = -1.2

Do these two expressions represent equivalent expressions? Explain why or why not. 36 + 20 4(9 + 5)

Answers

Answer:

yes because 4 times 9 is equal to 36  and 4 times 5 is equal 20

Step-by-step explanation:

Yes, both the expressions ( 36 + 20 ) and 4 ( 9 + 5 ) are equivalent to each other.

What is an expression?

Expression in maths is defined as the collection of the numbers variables and functions by using signs like addition, subtraction, multiplication, and division.

The given expressions are solved as:-

First expression,

36 + 20 = 56

Second expression,

4( 9 + 5 ) = 36 + 20

               = 56

From the above calculations,

( 36 + 20 ) = 4 ( 9 + 5 )

Therefore, both the expressions ( 36 + 20 ) and 4 ( 9 + 5 ) are equivalent to each other.

To know more about Expression follow

https://brainly.com/question/723406

#SPJ2

What is the equation of a line with a slope of –2 that passes through the point (6, 8)?

Answers

Answer:

The answer is

y = - 2x + 20

Step-by-step explanation:

To find the equation of the line given a point and slope we use the formula

[tex]y - y_{1} = m(x - x _{1})[/tex]

where

m is the slope

( x1 , y1) is the point

From the question

The point is (6, 8) and

slope - 2

The equation of the line is

[tex]y - 8 = - 2(x - 6) \\ y - 8 = - 2x + 12 \\ y = - 2x + 12 + 8[/tex]

We have the final answer as

y = - 2x + 20

Hope this helps you

Answer:

C on edge

Step-by-step explanation:

just took the quiz

Other Questions
Making sure that your business has something special and distinct to offer is known as? what is the best way to prevent cancer a - take prescribed medication as directed b - seek early genetic testing c - keep your vaccinations up to date d - avoid carcinogens Study the picture and tell me1) What do you think it is?2) When do you think it was used?3) Why it was made?4) How was it used?5) Who used it?) A bag contains 4 red marbles, 5 blue marbles, 6 yellow marbles, and 7 green marbles; Jonathan randomly grabs one marble, puts it back then draws a second marble. What is the probability that the first marble was red, and the second marble was green? Cheryl has $56 and wants to buy as many notebooks as she can to donate to her school. If each notebook costs $1.60,which inequality shows n, the maximum number of notebooks she can buy with her money?1.672 56O 1.6n s5656931.6O 56n2 1.6 Answers to the picture above What is 0.16666666666666666 as a fraction? Lorie is using long division to find the quotient of 13 + 6r2 + 5 and 12 + 1 1, as shown.41+ 5 12 + - 1I2 + 1 - 1) 1 3 + 612 + 0.1 + 5(13 + 12 1-5.12 + I + 5(5.12 + 51 5)-4r + 0What mistake did Lorie make?She combined the constants incorrectly in the last step.OB.The last term in the quotient has an incorrect denominator.. .She added an extra term to the dividend.OD.second term in the quotient should be 7. How many solutions can be found for the equation 3x 2 = 3x + 5? Zero One Two Infinitely Many HELP ME ANSWER THIS PLZ!!!!!! Which phrase from the text best supports the answer to Part A? Athena and Poseidon In a controlled experiment, a scientist is studying how long it takes parachutes of different sizes to fall to the ground. What is the experimental variable? Here is a list of numbers:11, 19, 2, 7, 8, 1,9,9,16,1State the median. Nathan read a total of 630 pages over 30 hours. In all, how many hours of reading will Nathan have to do this week in order to have read a total of 714 pages? Solve using unit rates. Which of the ten commandments did Proctor forget when reciting them to Hale? Thou shalt not kill. Thou shalt not steal. Thou shalt honor thy father and mother. Thou shalt not commit adultery. A(n) _____ describes a relationship between a noun or pronoun and another word in the sentence. are 18 and 2x like terms? Look at Tribute Money from the Brancacci Chapel. Which feature of thispainting contributes to the drama of the piece? please help me out with this Online servers that can be accessed through the internet are an example of RAM.O TrueO False